Application de l'homogénéisation

Olympiades mathématiques, énigmes et défis
Avatar de l’utilisateur
Olympus
Membre Irrationnel
Messages: 1668
Enregistré le: 12 Mai 2009, 12:00

par Olympus » 17 Juin 2010, 21:43

benekire2 a écrit:Une question :

Bon alors c'est pas a propos des transformations de ravi (je connais) mais a propos des inégalités. l'IAG je connais. Mais je lis souvent chez toi olympus des "AM-GM pondéré" c'est quoi au juste ?

Merci ;)


Regarde ici

^^



benekire2
Membre Transcendant
Messages: 4678
Enregistré le: 08 Avr 2009, 17:39

par benekire2 » 18 Juin 2010, 09:00

A, tu avais l'air de dire que fallait passer par jensen pour la prouver,

alors pour la pondérée je sais pas mais pour la version "normal" celle que j'appelle l'IAG, j'ai soit par récurrence , soit plus joli:

En notant A la moyenne arithmétique et G la géométrique et (a_i) ma famille :

donc il vient rapidement


Qui se réécrit:



i.e comme le membre de gauche vaut 1 :

et on conclu facilement que

Avatar de l’utilisateur
Olympus
Membre Irrationnel
Messages: 1668
Enregistré le: 12 Mai 2009, 12:00

par Olympus » 18 Juin 2010, 10:32

L'AM-GM se prouve par des méthodes plus softs que par la convexité, comme par l'inégalité du réordonnement, ou par la récurrence de Cauchy ( celle où on la montre pour les puissances de 2, puis on montre que si elle est vraie pour un N, alors elle l'est aussi pour N-1 ), ou encore par Muirhead ( c'est trivial comme la séquence majore ) . Et tous les cas particuliers se prouvent facilement par réécriture sous somme de carrés ^^

C'est plutôt l'AM-GM pondérée pour laquelle je n'ai pas trouvé de preuve élémentaire, à part passer par la convexité ou Jensen .

benekire2
Membre Transcendant
Messages: 4678
Enregistré le: 08 Avr 2009, 17:39

par benekire2 » 18 Juin 2010, 13:41

ba, pour l'AM-GM , la méthode que j'ai utilisé est celle de Polya, elle est vachement simple. Le problème d'utiliser muirhead là dessus c'est qu'il faut prouver muirhead.

Bref, cela ne résous pas l'AM-GM pondérée ...

manon_n
Membre Naturel
Messages: 40
Enregistré le: 23 Mar 2010, 19:25

par manon_n » 25 Juin 2010, 18:50

Ben314 a écrit:Allez, tient, juste pour voir, torche moi avec des complexes et toute la "géométrie invertive" (que je sais pas ce que c'est) que tu veut le petit problème suivant :
Soient C le cercle circonscrit à un triangle (non dégénéré) et C' son cercle inscrit.
Partant d'un point M de C, on construit les deux tangentes T1 et T2 à C'passant par M.
Ces deux tangentes recoupent C respectivement en N1 et N2.
Montrer que la droite (N1N2) est tangente à C'...

Juste pour dire que NON, la géométrie, contrairement aux inégalités, ça ne se résume pas et ça ne se résumera jamais à deux/trois recettes que l'on applique.


Bonjour,

je suis intéressée par cet exercice, mais je n'arrive pas a montrer que C' est le cercle circonscrit à MN1N2 , quelqu'un pourrait -il m'aider ?

Je vous remercie par avance.

Avatar de l’utilisateur
Ben314
Le Ben
Messages: 21535
Enregistré le: 11 Nov 2009, 22:53

par Ben314 » 02 Juil 2010, 23:50

Trés honètement, je ne connais pas de méthode simple pour résoudre cet exercice (c'est d'ailleur pour cela que je l'ai mis...) c'est en fait un cas particulier du "grand théorème de Poncelet".
Dans le cas particulier du triangle, je pense qu'une approche peut consister à commencer par montrer que, dans tout triangle, si O et I désignent respectivement les centre des cercles circonscrit et inscrit R et r leurs rayons, on a OI²=R²-2rR. La preuve la plus simple de ce résultat utilise les inversions.
On doit sans dout aussi pouvoir le faire avec des complexe, mais je pense que ça risque d'être fortement calculatoire...
Qui n'entend qu'un son n'entend qu'une sonnerie. Signé : Sonfucius

benekire2
Membre Transcendant
Messages: 4678
Enregistré le: 08 Avr 2009, 17:39

par benekire2 » 03 Juil 2010, 11:27

Salut à tous ,

J'avoue ne pas avoir trop cherché sur ce problème et donc rien trouvé ^^
Mais je pensais que c'était possible avec de la patiente pat les complexes en identifiant le cercle circonscrit au cercle unité. Mais c'est vraiment chaud j'ai l'impression.

Pour la démonstration de OI²=R²-2rR le plus simple c'est la puissance d'un point par rapport à un cercle. Bon, ok , c'est un peu de l'inversion ...

Bonne chance manon ^^

 

Retourner vers ⚔ Défis et énigmes

Qui est en ligne

Utilisateurs parcourant ce forum : Aucun utilisateur enregistré et 15 invités

Tu pars déja ?



Fais toi aider gratuitement sur Maths-forum !

Créé un compte en 1 minute et pose ta question dans le forum ;-)
Inscription gratuite

Identification

Pas encore inscrit ?

Ou identifiez-vous :

Inscription gratuite